Đến nội dung

iloveyouproht nội dung

Có 153 mục bởi iloveyouproht (Tìm giới hạn từ 29-04-2020)



Sắp theo                Sắp xếp  

#653973 gõ thử công thức toán

Đã gửi bởi iloveyouproht on 12-09-2016 - 23:37 trong Thử các chức năng của diễn đàn

Trước hết, ta chứng minh BĐT sau :
$$\dfrac{1}{\sqrt{a^2+bc}}+\dfrac{1}{\sqrt{b^2+ca}}+\dfrac{1}{\sqrt{c^2+ab}} \le \sqrt{2}\left (\dfrac{1}{a+b}+\dfrac{1}{b+c}+\dfrac{1}{c+a}\right )$$

Áp dụng CS, ta có :
$$\left (\sum \dfrac{1}{\sqrt{a^2+bc}}\right )^2 =\left [\sum \sqrt{\dfrac{(a+b)(a+c)}{a^2+bc}}.\dfrac{1}{\sqrt{(a+b)(a+c)}}\right ]^2 \le \left [\sum \dfrac{(a+b)(a+c)}{a^2+bc}\right ]\left [\sum \dfrac{1}{(a+b)(a+c)}\right ]$$
$$=\dfrac{2(a+b+c)}{(a+b)(b+c)(c+a)}.\left [\sum \dfrac{a(b+c)}{a^2+bc}+3\right ]$$
Như vậy, chỉ cần chứng minh :
$$\dfrac{2(a+b+c)}{(a+b)(b+c)(c+a)}.\left [\sum \dfrac{a(b+c)}{a^2+bc}+3\right ]\le 2\left (\sum \dfrac{1}{b+c}\right )^2$$
$$\Leftrightarrow \sum \dfrac{a(b+c)}{a^2+bc}+3\le \dfrac{\left (a^2+b^2+c^2+3ab+3bc+3ca\right )^2}{(a+b)(b+c)(c+a)(a+b+c)}$$
$$\Leftrightarrow \sum \dfrac{a(b+c)}{a^2+bc}-3 \le \dfrac{a^4+b^4+c^4-a^2b^2-b^2c^2-c^2a^2}{(a+b)(b+c)(c+a)(a+b+c)}$$
$$\Leftrightarrow \sum (a-b)(a-c)\left [\dfrac{1}{a^2+bc}+\dfrac{1}{(b+c)(a+b+c)}\right ] \ge 0$$
Không mất tính tổng quát, giả sử $a\ge b\ge c$, khi đó, ta có $a-c\ge \dfrac{a}{b}(b-c) \ge 0$
Do đó :
$$ \sum (a-b)(a-c)\left [\dfrac{1}{a^2+bc}+\dfrac{1}{(b+c)(a+b+c)}\right ] \ge \dfrac{(a-b)(a-c)}{b}\left \{a\left [\dfrac{1}{a^2+bc}+\dfrac{1}{(b+c)(a+b+c)}\right ]-b\left [\dfrac{1}{b^2+ca}+\dfrac{1}{(a+c)(a+b+c)}\right ]\right \}$$
$$=\dfrac{c(a-b)^2(a+b)(b-c)\left (a^2+b^2-ab+ac+bc\right )}{b(a+c)(b+c)(\left (a^2+bc\right )\left (b^2+ca\right )} \ge 0$$
Trở lại bài toán, ta chỉ cần chứng minh :
$$\sqrt{2}\left (\dfrac{1}{a+b}+\dfrac{1}{b+c}+\dfrac{1}{c+a}\right ) \le \dfrac{3(a+b+c)}{\sqrt{2}(ab+bc+ca)}$$
Thật vậy :
$$\dfrac{1}{a+b}+\dfrac{1}{b+c}+\dfrac{1}{c+a}\ =\dfrac{a^2+b^2+c^2+3(ab+bc+ca)}{(a+b)(b+c)(c+a)}$$
$$\le \dfrac{(a+b+c)^2+ab+bc+ca}{8\dfrac{(a+b+c)(ab+bc+ca)}{9}} \le \dfrac{3}{2}\dfrac{a+b+c}{ab+bc+ca}$$
$$\Leftrightarrow \sqrt{2}\left (\dfrac{1}{a+b}+\dfrac{1}{b+c}+\dfrac{1}{c+a}\right ) \le \dfrac{3(a+b+c)}{\sqrt{2}(ab+bc+ca)}$$
BĐT đã được chứng minh. Đẳng thức xảy ra khi $a=b=c$.

Qúy <3 Kiên




#662905 gõ thử công thức toán

Đã gửi bởi iloveyouproht on 24-11-2016 - 15:44 trong Thử các chức năng của diễn đàn

Bất đẳng thức tương đương :
 
$1+\frac{4\sum ab}{3+\sum ab} \ge 3\sqrt[3]{ \frac{8abc}{(a+1)(b+1)(c+1)}}$
Ta có :
 
$1+\frac{4\sum ab}{3+\sum ab} \ge 3 \sqrt[3]{\frac{4(\sum ab)^2}{(3+\sum ab)^2}}$. Đưa về cm :
 
$(\sum ab )^2 (a+1)(b+1)(c+1) \ge 2abc (3+\sum ab )^2$
 
$\Leftrightarrow (\sum ab )^2 ( \sum ab -abc +4)\ge 6abc(3+2\sum ab )$
 
mà $ (\sum ab )^2  \ge 3abc(a+b+c)=9abc$,=>
 
$ 3 ( \sum ab -abc +4)\ge 2 (3+2\sum ab )$
 
$\Leftrightarrow 12 \ge 3abc+2\sum ab$ ( right ) => 



#652509 gõ thử công thức toán

Đã gửi bởi iloveyouproht on 02-09-2016 - 23:42 trong Thử các chức năng của diễn đàn

5a . Đầu tiên ta chứng minh P $\leq 3$

    

Ta có : P-3=$\sum \frac{(a+b)^{2}}{\sum a^{2}+ab} -3 =\sum \frac{ab-c^{2}}{\sum a^{2}+ab} =\frac{(ab-c^{2})(3+\frac{1}{ab})}{(\sum a^{2}+ab)(1+1+1+\frac{1}{ab})} \leq \frac{3(\sum ab) -3(\sum c^{2})-\sum \frac{c^{2}}{ab}}{(a+b+c+1)^{2}}$ 

=> P-3 $\leq \frac{(a+b+c)^{2}+3-(a+b+c)^{2}-3}{(a+b+c+1)^{2}}=0$

=> P $\leq 3$ 

Dấu = tại a=b=c

Tiếp đến sẽ cm $\geq 2$ 

Ta có : $\sum \frac{(a+b)^{2}}{a^{2}+b^{2}+c^{2}+ab}\geq \sum \frac{(a+b)^{2}}{\sum a^{2}+\sum ab }=\frac{\sum (a+b)^{2}}{\sum a^{2}+\sum ab}=2$

Dấu = tại a=b=0 




#631755 cho a,b,c >0, chứng minh $\frac{a}{b+c}+...

Đã gửi bởi iloveyouproht on 07-05-2016 - 15:52 trong Bất đẳng thức và cực trị

\frac{a}{b+c} + \frac{b}{a+c} + \frac{c}{a+b}\geq \frac{3}{2}
=> \frac{a}{b+c}+ 1 + \frac{b}{a+c} +1 +  \frac{c}{a+b}+ 1\geq \frac{9}{2}
=> (a+b+c)(\frac{1}{b+c}+\frac{1}{a+c}+\frac{1}{a+b} \geq \frac{9}{2}
=> 2(a+b+c)(\frac{1}{b+c}+\frac{1}{a+c}+\frac{1}{a+b} \geq 9
Đặt a+b = x ; b+c=y ; c+a = z

=> (x+y+z)\frac{1}{x} + \frac{1}{y}+ \frac{1}{z} \geq 9

Ta có : \frac{x}{y} + \frac{y}{x} \geq  2

Nhân ra tự làm nhé :)))




#652758 CMR: $\sqrt{2a^{2}+ab+2b^{2}}+\s...

Đã gửi bởi iloveyouproht on 04-09-2016 - 15:28 trong Bất đẳng thức và cực trị

Cho a, b, c>0 thỏa mãn a+b+c=1

CMR: $\sqrt{2a^{2}+ab+2b^{2}}+\sqrt{2b^{2}+bc+2c^{2}}+\sqrt{2c^{2}+ca+2a^{2}}\geq \sqrt{5}$

Đối với những bài dạng như thế này , đầu tiên ta nhận thấy dấu = xảy ra tại a=b nên ta đưa nó về dạng : $\sqrt{(\alpha x+\beta y)^{2}+\gamma (x-y)^{2}}$ Để tìm Min

Mà : $\sqrt{(\alpha x+\beta y)^{2}+\gamma (x-y)^{2}} = \sqrt{(\alpha ^{2}+\gamma )x^{2}+(2\alpha \beta -2\gamma )xy+(\beta ^{2}+\gamma)y^{2} }$

Đồng nhất hệ số ta đưa nó về giải hệ : $$\left\{\begin{matrix} \alpha ^{2}+\gamma =2 \\2\alpha \beta -2\gamma =1 \\  \beta^{2}+\gamma=2\end{matrix}\right.$$

giải ra đươc : $\alpha =\beta =\sqrt{\frac{5}{2}} ; \gamma =\frac{3}{4}$

Sau đó đưa về cách giải của bạn le truong son đã đăng :D




#664734 Tìm GTNN của biểu thức: $\sqrt{a^2+ab+2b^2} +\sqrt{b^2+bc+2c^...

Đã gửi bởi iloveyouproht on 15-12-2016 - 20:17 trong Bất đẳng thức và cực trị

Cho a, b, c là các số thực không âm thỏa mãn điều kiện a + b + c = 1. Tìm GTNN của biểu thức: 

$\sqrt{a^2+ab+2b^2} +\sqrt{b^2+bc+2c^2} +\sqrt{c^2+ac+2a^2}$.

mong mọi người giải giùm.

Ở đây mình có cách giải tổng quát bài này chỉ bằng phép biến đổi tương đương . B xem tại đây : http://diendantoanho...ca2a2geq-sqrt5/




#660925 $a^{3} + b^{3} + c^{3} + 6abc \geq 9...

Đã gửi bởi iloveyouproht on 07-11-2016 - 00:16 trong Bất đẳng thức và cực trị

Cho a, b, c là các số thực không âm thỏa mãn điều kiện ab + bc + ca   = 3. Chứng minh rằng

$a^{3} + b^{3} + c^{3} + 6abc \geq 9$

Ta có : P= $a^{3} + b^{3} + c^{3} + 6abc$ => 2P =  $2(a^{3} + b^{3} + c^{3}) + 12abc$

Mà : $a^{3} + a^{3} + 4 \geq 6a^{2}$

Tương tự với b,c cộng lại ta được : $2P \geq  6(a^{2} + b^{2} + c^{2} + 12abc-12$ 

Ta có bđt : $a^{2} + b^{2} + c^{2} + 2abc \geq 2 ( ab + bc +ca )$ ( schur ) 

=> $2P \geq  6(a^{2} + b^{2} + c^{2} + 12abc-12 \geq 12( ab +bc +ca )-12=18$

=> đpcm 




#590596 Chứng minh $x^200+x^100+1$ chia hết cho $x^4 + x^2 + 1$

Đã gửi bởi iloveyouproht on 24-09-2015 - 02:48 trong Đại số

Chứng minh $x^200+x^100+1$ chia hết cho $x^4 + x^2 + 1$

Chứng minh $A = n^3 + (n+1)^3+(n+2)^3$ chia hết cho $9$ với mọi n :))




#590603 Chứng minh $x^200+x^100+1$ chia hết cho $x^4 + x^2 + 1$

Đã gửi bởi iloveyouproht on 24-09-2015 - 07:50 trong Đại số

a) Đặt $x^{2}=a$. Cần chứng minh: $a^{100}+a^{50} \vdots a^{2}+a+1$

Sử dụng tính chất quen thuộc: $a^{3m+1}+a^{3n+2} = a(a^{3m}-1) + a^{2}(a^{3n}-1) - (a^{2}+a+1) \vdots a^{2}+a+1$

b) $n^{3}+(n+1)^{3}+(n+2)^{3} = 3n^{3}+9n^{2}+15n+9= 3(x+1)(x^{2}+2x+3)$

Dễ thấy 1 trong 2 số $x+1$ và $x^{2}+2x$ chia hết cho 3.

Từ đó ta có đpcm.

cm $x^{200} +x^{100} + 1$  chia hết cho $x^4 + x^2 + 1$ ạ :))) Có fải cm $x^{200} +x^{100}$ chia hết cho $x^4 + x^2 + 1$ đâu ạ :))




#639011 Tìm max $(1+2a)(1+2bc)$

Đã gửi bởi iloveyouproht on 08-06-2016 - 21:23 trong Bất đẳng thức và cực trị

Đúng r . Cho xin luôn :)




#675525 Chứng minh rằng : $3(a^{2}+b^{2}+c^{2})+abc \geq 10$

Đã gửi bởi iloveyouproht on 28-03-2017 - 15:26 trong Bất đẳng thức và cực trị

Cho a,b,c là các số thực dương thỏa mãn $ab+bc+ca=3.$

Chứng minh rằng : $3(a^{2}+b^{2}+c^{2})+abc \geq 10$

Ta có bđt phụ : $\sum a^{2}+2abc+1\geq 2(\sum ab)$ (*)

Chứng minh (*): https://diendantoanh...bc1geq-2abbcca/

Áp dụng bất đẳng thức (*) ta có : $3(\sum a^{2})+abc=\frac{1}{2}\left [ 6(\sum a^{2})+2abc \right ]\geq \frac{1}{2}\left [ 5(\sum a^{2})+2(\sum ab) -1\right ]\geq \frac{1}{2}\left [ 7(\sum ab) -1\right ]=10$

-




#675710 Chứng minh rằng : $3(a^{2}+b^{2}+c^{2})+abc \geq 10$

Đã gửi bởi iloveyouproht on 30-03-2017 - 14:14 trong Bất đẳng thức và cực trị

đề là ab+bc+ca+abc=4 ạ, mình viết nhầm nữa ._.

Nếu gt thay đổi thì ta cũng chỉ cần biến đổi thêm chút là đc :v

Từ gt=>abc$\leq 1$

Ta có bđt phụ : $\sum a^{2}+2abc+1\geq 2(\sum ab)$ (*)

Chứng minh (*): https://diendantoanh...bc1geq-2abbcca/

Áp dụng bất đẳng thức (*) ta có : $3(\sum a^{2})+abc=\frac{1}{2}\left [ 6(\sum a^{2})+2abc \right ]\geq \frac{1}{2}\left [ 5(\sum a^{2})+2(\sum ab) -1\right ]\geq \frac{1}{2}\left [ 7(\sum ab) -1\right ]= \frac{1}{2}\left [ 7(4-abc) -1\right ]\geq 10$




#676062 cho 3 số thực dương x,y,z thỏa mãn

Đã gửi bởi iloveyouproht on 03-04-2017 - 00:40 trong Bất đẳng thức và cực trị

cho 3 số thực dương x,y,z thỏa mãn $x+y\leq z$ chứng minh rằng:$(x^{2}+y^{2}+z^{2})(\frac{1}{x^{2}}+\frac{1}{y^{2}}+\frac{1}{z^{2}})\geq \frac{27}{2}$

yaJ3lp3.png




#676151 Chứng minh rằng $(a+b-c-1)(b+c-a-1)(c+a-b-1)\leq 8$

Đã gửi bởi iloveyouproht on 04-04-2017 - 02:53 trong Bất đẳng thức và cực trị

Cho 3 số dương a,b,c thỏa mãn ab+bc+ca=abc,,,, :icon6:

Chứng minh rằng $(a+b-c-1)(b+c-a-1)(c+a-b-1)\leq 8$

 

Đây là đề thi tỉnh em hôm nay ,,đăng lên để mọi người tham khảo @@

Đào mộ :)

Đặt (x,y,z)->(a-1,b-1,c-1) => x,y,z>0

Ta có :

GT=> $\sum (x+1)(y+1)=\prod (x+1)<=>\sum x+2=xyz$

Ta cần cm : $\prod (a+b-c)\leq 8$

từ GT => $\sum x\geq 6$ => $xyz=\sum x+2\leq \frac{4(\sum x)}{3}$

Hay $\frac{xyz}{\sum x}\leq \frac{4}{3}$

Ta chỉ việc chứng minh : $\frac{xyz}{\sum x}\sqrt{\frac{27xyz}{\sum x}}\geq \prod (x+y-z)$

                           <=> $27x^{3}y^{3}z^{3}\geq (\sum x)^{3}\prod (x+y-z)^{2}$

Bây giờ lại đặt x+y-z=m ; y+z-x=n ; z+x-y=p => 2x=m+p ; 2y=m+n ; 2z=n+p 

 

Ta đưa bđt về cần cm : $27\prod (m+n)^{3}\geq 512m^{2}n^{2}p^{2}(\sum m)^{3}$

 Vì $9\prod (m+n)\geq 8( m+n+p)( mn+np+pm)$

Nên ta chỉ cần cm : $(m+n+p)^{3}( mn+np+pm)^{3}\geq 27m^{2}n^{2}p^{2}(m+n+p)^{3}$

              <=> $(mn+np+pm)^{3}\geq 27m^{2}n^{2}p^{2}$ ( đúng theo cauchy )

Dấu '=' xảy ra khi a=b=c=3




#656675 $\sum \frac{4}{a+b} \leq \frac{1}{a}+\frac{1}{b...

Đã gửi bởi iloveyouproht on 04-10-2016 - 17:00 trong Bất đẳng thức và cực trị

Bài 1: Cho $a,b,c$ là các số thực dương thoả mãn $abc = 1$.Chứng minh rằng:

        $a^4 + b^4 +b^4 + a + b + c + \frac{2a}{b^2+c^2} + \frac{2b}{a^2+c^2} + \frac{2c}{a^2+b^2} \geq 9$

Bài 2: Cho $a,b,c$ là các số thực dương tuỳ ý. Chứng minh rằng:

        $\sqrt{5a^2+4bc} + \sqrt{5b^2+4ca} + \sqrt{5c^2+4ab} \geq \sqrt{3(a^2+b^2+c^2)} + 2(\sqrt{ab}+\sqrt{bc}+\sqrt{ca})$

Bài 3: Cho $a,b,c$ là các số thực dương thoả mãn $a+b+c = 1$. Chứng minh rằng:

        $\frac{4}{a+b}+\frac{4}{b+c}+\frac{4}{c+a} \leq \frac{1}{a}+\frac{1}{b}+\frac{1}{c}+9$

Bài 4: Cho $a,b,c$ là các số thực dương thoả mãn $ab+bc+ca = 1$. Chứng minh rằng:

        $\frac{a^3}{1+9b^2ca}+\frac{b^3}{1+9c^2ab}+\frac{c^3}{1+9a^2bc} \geq \frac{(a+b+c)^3}{18}$

Bài 5: Cho $a,b,c$ phân biệt. Chứng minh rằng: 

        $(a^2+b^2+c^2)\left ( \frac{1}{(a-b)^2}+\frac{1}{(b-c)^2}+\frac{1}{(c-a)^2}\right ) \geq \frac{9}{2}$

Bài 6: Cho $a,b,c$ là các số thực dương thoả mãn $2ab+6bc+2ac=7abc$

     Tìm GTNN của $C=\frac{4ab}{a+2b}+\frac{9ca}{a+4c}+\frac{4bc}{b+c}$

Bài 7: Cho $a,b,c$ là các số thực dương thoả mãn $2\left ( \frac{a}{b}+\frac{b}{a}\right )+c\left ( \frac{a}{b^2}+\frac{b}{a^2} \right )=6$.

     Tìm GTNN của $P=\frac{bc}{a(2b+c)}+\frac{ca}{b(2a+c)}+\frac{4ab}{c(a+b)}$

Bài 8: Cho 3 sô thực dương $x,y,z$ thoả mãn $x+y+z \leq \frac{3}{2}$.

     Tìm GTNN của $P=\frac{x(yz+1)^2}{z^2(zx+1)}+\frac{y(zx+1)^2}{x^2(xy+1)}+\frac{z(xy+1)^2}{y^2(yz+1)}$

Bài 9: Cho các số thực dương thoả mãn $\sqrt{a}+\sqrt{b}+\sqrt{c}=1$.

     Tìm GTNN của $P=\sqrt{2a^2+ab+2b^2}+\sqrt{2b^2+bc+2c^2}+\sqrt{2c^2+ca+2a^2}$.

Mọi người giúp em với ạ. Em xin cảm ơn!!!

Bài 2 đã được anh dogsteven Giari . Mình xin trích lại như sau : 

Bất đẳng thức có tích rời rạc, việc đầu tiên của ta là gom lại.

Bất đẳng thức trên tương đương với: $\sum \dfrac{5a^2}{\sqrt{5a^2+4bc}+2\sqrt{bc}}\geqslant \sqrt{3(a^2+b^2+c^2)}$

Áp dụng bất đẳng thức Cauchy-Schwarz: $VT\geqslant \dfrac{5(a^2+b^2+c^2)^2}{\sum a^2\sqrt{5a^2+4bc}+2\sum a^2\sqrt{bc}}$

Tiếp theo là "phá căn". Áp dụng bất đẳng thức Cauchy-Schwarz:

$\sum a^2\sqrt{5a^2+4bc}\leqslant \sqrt{(a^2+b^2+c^2)\left[5(a^4+b^4+c^4)+4abc(a+b+c)\right]}$

$2\sum a^2\sqrt{bc}\leqslant \dfrac{2}{\sqrt{3}}\sqrt{a^2+b^2+c^2}(ab+bc+ca)$

Do đó ta chỉ cần chứng minh: $5(a^2+b^2+c^2)\geqslant \sqrt{15(a^4+b^4+c^4)+12(ab+bc+ca)}+2(ab+bc+ca)$

Đến đây dễ rồi.




#647052 Cho x, y, z>0 : CMR: $\sum \frac{x^{3}...

Đã gửi bởi iloveyouproht on 29-07-2016 - 16:55 trong Bất đẳng thức và cực trị

Cho x, y, z>0 :

CMR: $\frac{x^{3}}{(x+y)^{3}}+\frac{y^{3}}{(y+z)^{3}}+\frac{z^{3}}{(z+x)^{3}}\geq \frac{3}{8}$

 

Áp dụng bđt holder ta có : $(\sum \frac{x^{3}}{(x+y)^{3}} )9\geq (\sum \frac{x}{x+y})^{3} => VT\geq \frac{1}{9}(\sum \frac{x}{x+y})^{3} Ta cần cm : \frac{1}{9}(\sum \frac{x}{x+y})^{3} \geq \frac{3}{8} <=> \sum \frac{x}{x+y} \geq \frac{3}{2} Đến đây b tự làm nha$




#633507 CM: $\sqrt{\frac{a}{b+c}} + \sqrt{\frac{b}{a+c}} +...

Đã gửi bởi iloveyouproht on 16-05-2016 - 18:34 trong Bất đẳng thức và cực trị

Cho a,b,c dương Chứng minh : 

$\sqrt{\frac{a}{b+c}} + \sqrt{\frac{b}{a+c}} + \sqrt{\frac{c}{a+b}} \geq 2$

Cho mình biết dấu = xảy ra khi nào vậy nhé :))




#663282 UKMO 2005

Đã gửi bởi iloveyouproht on 28-11-2016 - 15:11 trong Bất đẳng thức và cực trị

Sử dụng AM-GM :  :D

 

(\frac{a}{b}+\frac{b}{c}+\frac{c}{a})^{2}\geq (a+b+c)(\frac{1}{a}+\frac{1}{b}+\frac{1}{c})

Cách khác nếu b cần :)

Ta có :

$(\frac{a}{b}+\frac{b}{c}+\frac{c}{a})(ab+bc+ca)\geq (a+b+c)^{2} <=> (\frac{a}{b}+\frac{b}{c}+\frac{c}{a})abc(\frac{1}{a}+\frac{1}{b}+\frac{1}{c})\geq (a+b+c)^{2}$

 

 

$(\frac{a}{b}+\frac{b}{c}+\frac{c}{a})(\frac{1}{ab}+\frac{1}{bc}+\frac{1}{ca})\geq (\frac{1}{a}+\frac{1}{b}+\frac{1}{c})^{2}$

Nhần vế theo vế ta được :

$(\frac{a}{b}+\frac{b}{c}+\frac{c}{a})abc(\frac{1}{a}+\frac{1}{b}+\frac{1}{c})(\frac{a}{b}+\frac{b}{c}+\frac{c}{a})(\frac{1}{ab}+\frac{1}{bc}+\frac{1}{ca})\geq (a+b+c)^{2}(\frac{1}{a}+\frac{1}{b}+\frac{1}{c})^{2} <=>(\frac{a}{b}+\frac{b}{c}+\frac{c}{a})^{2}(\frac{1}{a}+\frac{1}{b}+\frac{1}{c})(a+b+c)\geq (a+b+c)^{2}(\frac{1}{a}+\frac{1}{b}+\frac{1}{c})^{2}$

=> Đpcm 




#686638 Tuyển tập tính chất trong toán hình học phẳng

Đã gửi bởi iloveyouproht on 05-07-2017 - 22:11 trong Chuyên đề toán THCS

Tài liệu này dc úp trên trang lttk mà giá mua là 50k khá chát. Nên mình đóng thành pdf  để mọi người in và dùng free :)). Nhớ like và share nếu thấy tài liệu bổ ích :)

https://drive.google...aXZrbmlHYXkxVVk

tks thím <3




#647502 $(a^{2}+b^{2}+c^{2})^{3} \g...

Đã gửi bởi iloveyouproht on 01-08-2016 - 16:27 trong Bất đẳng thức và cực trị

Em dung hệ số bất định. Khi triển bất đẳng thức tổng quát bên dưới rồi đồng nhất hệ số với bất đẳng thức cần minh. Bản chất của việc này là giải một hệ phương trình tuyến tính.

Ah có tài liệu gì về pp này không ạ ! E nghe k hiểu ý :P




#647447 $(a^{2}+b^{2}+c^{2})^{3} \g...

Đã gửi bởi iloveyouproht on 01-08-2016 - 02:11 trong Bất đẳng thức và cực trị

Ta thuần nhất bất đẳng thức lại dưới dạng

\[(a^{2}+b^{2}+c^{2})^{3} \geqslant 9abc(a^{3}+b^{3}+c^{3}),\]

hay là

\[\sum \left [a^4+11c^4+6b^3c+2ab^3+4ca^3+3c^2a^2+2(a^2-bc)^2+3(b^2-ca)^2+(c^2-ab)^2  \right ](a-b)^2 \geqslant 0.\]

Hiển nhiên đúng.

Ah có thủ thuật gì khi phân tích được ra như thế không ạ




#668887 Tìm min $\sum \frac{x^4}{(x^2+y^2)(x+y)}$

Đã gửi bởi iloveyouproht on 19-01-2017 - 20:51 trong Bất đẳng thức và cực trị

chung minh voi x,y,z la cac so thuc duong sao cho x+y+z=1 tim gia tri nho nhat cua

$\frac{x^4}{(x^2+y^2)(x+y)}+\frac{y^4}{(y^2+z^2)(y+z)}+\frac{z^4}{(z^2+x^2)(z+x)}$ :wacko: :wub: :luoi: :icon6: :icon10:  :icon10:  :icon10:  :wub:  :wub:  :wub:

:$\sum \frac{x^4}{(x+y)(x^2+y^2)}-\sum \frac{y^4}{(x+y)(x^2+y^2)}=\sum x-\sum y=0$

=>$\sum \frac{x^4}{(x+y)(x^2+y^2)}=\frac{1}{2}(\sum \frac{x^4}{(x+y)(x^2+y^2)}+\sum \frac{y^4}{(x+y)(x^2+y^2)})=\frac{1}{2} \sum \frac{x^4+y^4}{(x+y)(x^2+y^2)}\geq \frac       {1}{4}\sum \frac{(x^2+y^2)^2}{(x+y)(x^2+y^2)}\geq \frac{1}{8}\sum \frac{ (x+y)^2}{x+y}=\frac{1}{8}\sum (x+y)=\frac{1}{4}$

=>đpcm




#647116 Tìm Max $P=ac+bd+cd$

Đã gửi bởi iloveyouproht on 29-07-2016 - 22:32 trong Bất đẳng thức và cực trị

THCS chưa học khảo sát hàm anh nhé!

Có cách nào khác không b :D




#648434 tìm GTNN của $A= 3^x+3^y$ với x+y=4

Đã gửi bởi iloveyouproht on 07-08-2016 - 17:24 trong Bất đẳng thức và cực trị

1.tìm GTLN của B= ($(\sqrt{a}+ \sqrt{b})^{4} +(\sqrt{a}+\sqrt{c})^{4} +(\sqrt{a}+\sqrt{d})^{4}+(\sqrt{b}+\sqrt{d})^{4}+(\sqrt{b}+\sqrt{c})^{4}+(\sqrt{c}+\sqrt{d})^{4}$ với a+b+c+d=1 và a,b,c,d>0

 

 

B xem lại đề dùm mình được không ? Hình như đúng phải là : $\sum (\sqrt{a}-\sqrt{b})^{4}$

Ta có : $(\sqrt{a}-\sqrt{b})^{4}=(a+b-2\sqrt{ab})^{2}=(a+b)^{2} -4(a+b)\sqrt{ab} + 4ab \leq (a+b)^{2} - 4ab = a^{2}+b^{2}-2ab$

Tương Tự ta được : $\sum (\sqrt{a}-\sqrt{b})^{4} \leq 3(a+b+c+d)^{2}-8(ab+ac+ad+bd+bc+cd)\leq 3(a+b+c+d)^{2}=3$

Vậy max B =3 

Dấu = xảy ra khi (a;b;c;d)=(1;0;0;0) và các hoán vị




#647218 $4\sqrt{x+1}+xy\sqrt{y^{2}+4}=0...

Đã gửi bởi iloveyouproht on 30-07-2016 - 19:56 trong Phương trình - hệ phương trình - bất phương trình

pt (1) <=> $2y^{2} - 9y +4 = \frac{4}{x} +2

<=> (2y^{2}-9y+4)^{2}=\frac{16}{x^{2}+\frac{16}{x}+4}

<=> \frac{16}{x^{2}} + \frac{16}{x} = (2y^{2}-9y+4)^{2} -4$

Do x khác 0 chia phương trình (2) choa x ta được : $4\sqrt{x+1} = -y\sqrt{y^{2}+4}

$=> y^{4} +4y^{2} = (2y^{2}-9y+4)^{2} -4 <=> (y^{2}+2)^{2} = (2y^{2}-9y+4)^{2} => y^{2}+2=\pm (2y^{2}-9y+4) =>y=>x .....$

 

Giải hệ phương trình:

$\left\{\begin{matrix} &2y^{2}-9y-\dfrac{4}{x}=-2 \\ &4\sqrt{x+1}+xy\sqrt{y^{2}+4}=0 \end{matrix}\right.$

 

<=> \frac{16}{x^{2}} +\frac{16}{x} = y^{2}(y^{2}+4)$